Saltar al contenido principal
LibreTexts Español

5.3: CAPÍTULO 3

  • Page ID
    107846
  • \( \newcommand{\vecs}[1]{\overset { \scriptstyle \rightharpoonup} {\mathbf{#1}} } \) \( \newcommand{\vecd}[1]{\overset{-\!-\!\rightharpoonup}{\vphantom{a}\smash {#1}}} \)\(\newcommand{\id}{\mathrm{id}}\) \( \newcommand{\Span}{\mathrm{span}}\) \( \newcommand{\kernel}{\mathrm{null}\,}\) \( \newcommand{\range}{\mathrm{range}\,}\) \( \newcommand{\RealPart}{\mathrm{Re}}\) \( \newcommand{\ImaginaryPart}{\mathrm{Im}}\) \( \newcommand{\Argument}{\mathrm{Arg}}\) \( \newcommand{\norm}[1]{\| #1 \|}\) \( \newcommand{\inner}[2]{\langle #1, #2 \rangle}\) \( \newcommand{\Span}{\mathrm{span}}\) \(\newcommand{\id}{\mathrm{id}}\) \( \newcommand{\Span}{\mathrm{span}}\) \( \newcommand{\kernel}{\mathrm{null}\,}\) \( \newcommand{\range}{\mathrm{range}\,}\) \( \newcommand{\RealPart}{\mathrm{Re}}\) \( \newcommand{\ImaginaryPart}{\mathrm{Im}}\) \( \newcommand{\Argument}{\mathrm{Arg}}\) \( \newcommand{\norm}[1]{\| #1 \|}\) \( \newcommand{\inner}[2]{\langle #1, #2 \rangle}\) \( \newcommand{\Span}{\mathrm{span}}\)\(\newcommand{\AA}{\unicode[.8,0]{x212B}}\)

    Ejercicio\(3.1.6\).

    Contestar

    1. Observe que cuando\(x\) está cerca\(1 / 2\),\(f(x)\) está cerca\(1 / 2\) no importa si\(x\) es racional o irracional. Tenemos\ [|f (x) -1/2|=\ left\ {\ begin {array} {ll}
      |x-1/2|, &\ text {if} x\ in\ mathbb {Q}\ text {;}\\
      |1-x-1/2|, &\ text {if} x\ notin\ mathbb {Q}\ text {.}
      \ end {array}\ right.\] Así,\(|f(x)-1 / 2|=|x-1 / 2|\) para todos\(x \in \mathbb{R}\).

    Dado cualquiera\(\varepsilon > 0\), elija\(\delta = \varepsilon\). Entonces\[|f(x)-1 / 2|<\varepsilon \text { whenever }|x-1 / 2|<\delta .\] Por lo tanto,\(\lim _{x \rightarrow 1 / 2} f(x)=1 / 2\).

    1. Observe que cuando\(x\) está cerca\(0\) y\(x\) es racional,\(f(x)\) está cerca\(0\). No obstante, cuando\(f\) está cerca\(0\) y\(x\) es irracional,\(f(x)\) está cerca\(1\). Así, el límite dado no existe. Esto lo justificamos utilizando el criterio secuencial para los límites (Teorema 3.1.2). Por contradicción, supongamos que\[\lim _{x \rightarrow 0} f(x)=\ell ,\] donde\(\ell\) está un número real. Elija una secuencia\(\left\{r_{n}\right\}\) de números racionales que converja a\(0\). Entonces\(f\left(r_{n}\right)=r_{n}\) y\(f\left(s_{n}\right)=1-s_{n}\) y, de ahí,\[\ell=\lim _{n \rightarrow \infty} f\left(r_{n}\right)=0\] y\[\ell=\lim _{n \rightarrow \infty} f\left(s_{n}\right)=\lim _{n \rightarrow \infty}\left(1-s_{n}\right)=1 .\] Esto es una contradicción.
    2. Por un método similar a la parte (b), podemos demostrar que\(\lim _{x \rightarrow 1} f(x)\) no existe.

    Resolver este problema sugiere un problema más general de la siguiente manera. Dados dos polinomios\(P\) y\(Q\), definir la función\ [f (x) =\ left\ {\ begin {array} {ll}
    P (x), &\ text {if} x\ in\ mathbb {Q}\ text {;}\\
    Q (x), &\ text {if} x\ notin\ mathbb {Q}\ text {.}
    \ end {array}\ right.\] Si\(a\) es una solución de la ecuación\(P(x)=Q(x)\), es decir\(P(a)=Q(a)\), entonces el límite\(\lim _{x \rightarrow a} f(x)\) existe y el límite es este valor común. Para todos los demás puntos el límite no existe.

    Problemas similares:

    1. \(a \in \mathbb{R}\)Determina todo en el que\(\lim _{x \rightarrow a} f(x)\) existe, donde\ [f (x) =\ left\ {\ begin {array} {ll}
      x^ {2}, &\ text {if} x\ in\ mathbb {Q}\ text {;}\\
      x+2, &\ text {if} x\ in x\ notin\ mathbb {Q}\ text {.}
      \ end {array}\ derecho.\]
    2. Considera la función\ [f (x) =\ left\ {\ begin {array} {ll}
      x^ {2} +1, &\ text {if} x\ in\ mathbb {Q}\ text {;}\\
      -x, &\ text {if} x\ notin\ mathbb {Q}\ text {.}
      \ end {array}\ derecho.\]

    Demostrar que\(f\) no tiene límite en ningún caso\(a \in \mathbb{R}\).

    Ejercicio\(3.2.5\).

    Contestar

    La condición dada implica que si ambos\(x_{1}\) y\(x_{2}\) están cerca\(\bar{x}\), entonces están cerca el uno del otro y, por lo tanto,\(f\left(x_{1}\right)\) y\(f\left(x_{2}\right)\) están cerca el uno del otro. Esto sugiere el uso del criterio de Cauchy de límite para resolver el problema. Dado cualquiera\(\varepsilon > 0\), elija\(\delta=\frac{\varepsilon}{2(k+1)}\). Si\(x_{1}, x_{2} \in D \backslash\{\bar{x}\}\) con\(\left|x_{1}-\bar{x}\right|<\delta\) y\(\left|x_{2}-\bar{x}\right|<\delta\), entonces\[\left|f\left(x_{1}\right)-f\left(x_{2}\right)\right| \leq k\left|x_{1}-x_{2}\right| \leq k\left(\left|x_{1}-\bar{x}\right|+\left|x_{2}-\bar{x}\right|\right)<k(\delta+\delta)=2 k \frac{\varepsilon}{2(k+1)}<\varepsilon .\] Por lo tanto,\(\lim _{x \rightarrow \bar{x}} f(x)\) existe.

    Ejercicio\(3.3.8\).

    Contestar

    1. Observe eso\(f(a)=g(a)=h(a)\) y, por lo tanto,\ [|f (x) -f (a) |=\ left\ {\ begin {array} {ll}
      |g (x) -g (a) |, &\ text {if} x\ in\ mathbb {Q}\ cap [0,1]\ text {;}\\
      |h (x) -h (a) |, &\ text {if} x\ in\ mathbb {Q} ^ {c}\ cap [0,1]\ texto {.}
      \ end {array}\ right.\] De ello se deduce que\[|f(x)-f(a)| \leq|g(x)-g(a)|+|h(x)-h(a)| \text { for all } x \in[0,1] .\] Por lo tanto,\(\lim _{x \rightarrow a} f(x)=f(a)\) y, así,\(f\) es continuo en\(a\).
    2. Aplicar la parte (a).

    Ejercicio\(3.3.9\).

    Contestar

    En cualquier número irracional\(a \in (0, 1]\), tenemos\(f(a) = 0\). Si\(x\) está cerca\(a\) y\(x\) es irracional, es obvio que\(f(x) = 0\) está cerca\(f(a)\). En el caso cuando\(x\) está cerca\(a\) y\(x\) es racional,\(f(x)=1 / q\) dónde\(p, q \in \mathbb{N}\). Veremos en la parte (a) que para cualquiera\(\varepsilon > 0\), sólo hay un número finito de\(x \in (0, 1]\) tal que\(f(x) \geq \varepsilon\). Así que\(f(x)\) está cerca\(f(a)\) para todos\(x \in (0, 1]\) excepto por un número finito de\(x \in \mathbb{Q}\). Ya que\(a\) es irracional, podemos elegir un barrio suficientemente pequeño de\(a\) para anular tal\(x\).

    1. Para cualquier\(\varepsilon > 0\),\[A_{\varepsilon}=\{x \in(0,1]: f(x) \geq \varepsilon\}=\left\{x=\frac{p}{q} \in \mathbb{Q}: f(x)=\frac{1}{q} \geq \varepsilon\right\}=\left\{x=\frac{p}{q} \in \mathbb{Q}: q \leq \frac{1}{\varepsilon}\right\} .\] Claramente, el número de\(q \in \mathbb{N}\) tales que\(q \leq \frac{1}{\varepsilon}\) es finito. Ya que\(0<\frac{p}{q} \leq 1\), tenemos\(p \leq q\). Por lo tanto,\(A_{\varepsilon}\) es finito.
    2. Arreglar cualquier número irracional\(a \in (0, 1]\). Entonces\(f(a) = 0\). Dado cualquiera\(\varepsilon > 0\), por la parte (a), el conjunto\(A_{\varepsilon}\) es finito, así que podemos escribir\[A_{\varepsilon}=\{x \in(0,1]: f(x) \geq \varepsilon\}=\left\{x_{1}, x_{2}, \ldots, x_{n}\right\} ,\] para algunos\(n \in \mathbb{N}\), donde\(x_{i} \in \mathbb{Q}\) para todos\(i=1, \ldots, n\). Ya que\(a\) es irracional, podemos elegir\(\delta > 0\) tal que\(x_{i} \notin(a-\delta, a+\delta)\) para todos\(i=1, \ldots, n\) (más precisamente, podemos elegir\(\delta=\min \left\{\left|a-x_{i}\right|: i=1, \ldots, n\right\}\)). Entonces\[|f(x)-f(a)|=f(x)<\varepsilon \text { whenever }|x-a|<\delta .\] Por lo tanto,\(f\) es continuo en\(a\).

    Ahora arregle cualquier número racional\(b=\frac{p}{q} \in(0,1]\). Entonces\(f(b)=\frac{1}{q}\). Elija una secuencia de números irracionales\(\left\{s_{n}\right\}\) que converja a\(b\). Ya que\(f\left(s_{n}\right)=0\) para todos\(n \in \mathbb{N}\), la secuencia\(\left\{f\left(s_{n}\right)\right\}\) no converge a\(f(b)\). Por lo tanto, no\(f\) es continuo en\(b\).

    En este problema, consideramos que el dominio de\(f\) es el intervalo\((0, 1]\), pero la conclusión sigue siendo válida para otros intervalos. En particular, podemos mostrar que la función definida\(\mathbb{R}\) por\ [f (x) =\ left\ {\ begin {array} {ll}
    \ frac {1} {q}, &\ text {if} x=\ frac {p} {q}, p, q\ in\ mathbb {N},\ text {where} p\ text {y} q\ text {no tienen factores comunes;}\
    1, &\ texto {si} x=0\ texto {;}\\
    0, &\ text {if} x\ text {es irracional}\ text {,}
    \ end {array}\ right.\] es continuo en cada punto irracional, y discontinuo en cada punto racional.

    Ejercicio\(3.3.10\).

    Contestar

    Considerar\ [f (x) =\ left\ {\ begin {array} {ll}
    \ left (x-a_ {1}\ right)\ left (x-a_ {2}\ right)\ cdots\ left (x-a_ {k}\ right), &\ text {if} x\ in\ mathbb {Q}\ text {;}\\
    0, &\ text {if} x\ in\ mathbb {Q} ^ {c}\ texto {.}
    \ end {array}\ derecho.\]

    Ejercicio\(3.4.6\).

    Contestar

    Dejar\(\alpha=\min \{f(x): x \in[a, b]\}\) y\(\beta=\max \{f(x): x \in[a, b]\}\). Entonces\[\frac{f\left(x_{1}\right)+f\left(x_{2}\right)+\cdots+f\left(x_{n}\right)}{n} \leq \frac{n \beta}{n}=\beta .\] Similarmente,\[\alpha \leq \frac{f\left(x_{1}\right)+f\left(x_{2}\right)+\cdots+f\left(x_{n}\right)}{n} .\] Entonces la conclusión se desprende del Teorema del Valor Intermedio.

    Ejercicio\(3.4.7\).

    Contestar

    1. Observe que\[|f(1 / n)| \leq 1 / n \text { for all } n \in \mathbb{N} .\]
    2. Aplicar el Teorema de Valor Extremo para la función\(g(x)=\left|\frac{f(x)}{x}\right|\) en el intervalo\([a, b]\).

    Ejercicio\(3.4.8\).

    Contestar

    Primero consideremos el caso donde\(f\) es monótona decreciente en\([0, 1]\). Por el Ejercicio 3.4.5,\(f\) tiene un punto fijo en\([0, 1]\), lo que significa que existe\(x_{0} \in[0,1]\) tal que\[f\left(x_{0}\right)=x_{0}\] Since\(f\) es monótona decreciente,\(f\) tiene un punto fijo único. En efecto, supongamos que existe\(x_{1} \in[0,1]\) tal que\(f\left(x_{1}\right)=x_{1}\). Si\(x_{1}<x_{0}\), entonces\(x_{1}=f\left(x_{1}\right) \geq f\left(x_{0}\right)=x_{0}\), lo que produce una contradicción. Es similar para el caso donde\(x_{1}>x_{0}\). Por lo tanto,\(x_{0}\) es el punto único en\([0, 1]\) tal que\(f\left(x_{0}\right)=x_{0}\).

    Ya que\(f(g(x))=g(f(x))\) para todos\(x \in [0, 1]\), tenemos\[f\left(g\left(x_{0}\right)\right)=g\left(f\left(x_{0}\right)\right)=g\left(x_{0}\right) .\] Así, también\(g\left(x_{0}\right)\) es un punto fijo de\(f\) y, de ahí,\(g\left(x_{0}\right)=x_{0}=f\left(x_{0}\right)\). El comprobante está completo en este caso.

    Considera el caso donde\(f\) está aumentando monótona. En este caso,\(f\) podría tener varios puntos fijos sobre\([0, 1]\), por lo que el argumento anterior no funciona. No obstante, por el Ejercicio 3.4.5, existe\(c \in [0, 1]\) tal que\(g(c) = c\). Defina la secuencia de la\(\left\{x_{n}\right\}\) siguiente manera:\ [\ begin {aligned}
    x_ {1} &=c\ text {,}\\
    x_ {n+1} &=f\ left (x_ {n}\ right)\ text {para todos} n\ geq 1\ text {.}
    \ end {aligned}\] Dado que\(f\) es monótona creciente,\(\left\{x_{n}\right\}\) es una secuencia monótona. De hecho, si\(x_{1} \leq x_{2}\), entonces\(\left\{x_{n}\right\}\) es monótona creciente; si\(x_{1} \geq x_{2}\), entonces\(\left\{x_{n}\right\}\) es monótona decreciente. Ya que\(f\) está acotado, por el teorema de convergencia monótona (Teorema 2.3.1), existe\(x_{0} \in[0,1]\) tal que\[\lim _{n \rightarrow \infty} x_{n}=x_{0} .\] Desde\(f\) es continuo y\(x_{n+1}=f\left(x_{n}\right)\) para todos\(n \in \mathbb{N}\), tomando límites tenemos\(f\left(x_{0}\right)=x_{0}\).

    Podemos demostrar por inducción que\(g\left(x_{n}\right)=x_{n}\) para todos\(n \in \mathbb{N}\). Entonces\[g\left(x_{0}\right)=\lim _{n \rightarrow \infty} g\left(x_{n}\right)=\lim x_{n}=x_{0} .\] Por lo tanto,\(f\left(x_{0}\right)=g\left(x_{0}\right)=x_{0}\).

    Ejercicio\(3.5.2\).

    Contestar

    1. Vamos\(f: D \rightarrow \mathbb{R}\). Del Teorema 3.5.3 vemos que si existen dos secuencias\(\left\{x_{n}\right\}\) y\(\left\{y_{n}\right\}\) en\(D\) tal que\(\left|x_{n}-y_{n}\right| \rightarrow 0\) como\(n \rightarrow \infty\), pero\(\left\{\left|f\left(x_{n}\right)-f\left(y_{n}\right)\right|\right\}\) no converge a\(0\), entonces no\(f\) es uniformemente continuo en\(D\). En términos generales,\(f\) para que sean uniformemente continuos en\(D\), si\(x\) y\(y\) están cerca el uno del otro, entonces\(f(x)\) y\(f(y)\) deben estar cerca el uno del otro. El comportamiento de la gráfica de la función de cuadratura sugiere el argumento a continuación para mostrar que no\(f(x)=x^{2}\) es uniformemente continuo en\(\mathbb{R}\).

    Definir dos secuencias\(\left\{x_{n}\right\}\) y de la\(\left\{y_{n}\right\}\) siguiente manera:\(x_{n}=n\) y\(y_{n}=n+\frac{1}{n}\) para\(n \in \mathbb{N}\). Entonces\(\left|x_{n}-y_{n}\right|=\frac{1}{n} \rightarrow 0\) como\(n \rightarrow \infty\). Sin embargo,\[\left|f\left(x_{n}\right)-f\left(y_{n}\right)\right|=\left(n+\frac{1}{n}\right)^{2}-n^{2}=2+\frac{1}{n^{2}} \geq 2 \text { for all } n \in \mathbb{N} .\] por lo tanto,\(\left\{\left|f\left(x_{n}\right)-f\left(y_{n}\right)\right|\right\}\) no converge\(0\) y, por lo tanto, no\(f\) es uniformemente continuo en\(\mathbb{R}\). En esta solución, podemos usar\(x_{n}=\sqrt{n+\frac{1}{n}}\) y\(y_{n}=\sqrt{n}\) para\(n \in \mathbb{N}\) su lugar.

    1. Uso\(x_{n}=\frac{1}{\pi / 2+2 n \pi}\) y\(y_{n}=\frac{1}{2 n \pi}\),\(n \in \mathbb{N}\).
    2. Uso\(x_{n}=1 / n\) y\(y_{n}=1 /(2 n)\).

    Es natural preguntarse si la función\(f(x)=x^{3}\) es uniformemente continua en\(\mathbb{R}\). Siguiendo la solución para la parte (a), podemos usar\(x_{n}=\sqrt[3]{n+\frac{1}{n}}\) y\(y_{n}=\sqrt[3]{n}\)\(n \in \mathbb{N}\) para demostrar que no\(f\) es uniformemente continuo en\(\mathbb{R}\). Por un método similar, podemos demostrar que la función\(f(x)=x^{n}\),\(n \in \mathbb{N}\),\(n \geq 2\), no es uniformemente continua en\(\mathbb{R}\). Una pregunta más desafiante es determinar si un polinomio de grado mayor o igual a dos es uniformemente continuo\(\mathbb{R}\).

    Ejercicio\(3.5.7\).

    Contestar

    Pista: Para la parte (a) usa Teorema 3.5.5. Para la parte (b) demostrar que la función se puede extender a una función continua en\([a, b]\) y luego usar el Teorema 3.5.5.

    Ejercicio\(3.5.8\).

    Contestar

    1. Aplicando la definición de límite, encontramos\(b > a\) tal que\[c-1<f(x)<c+1 \text { whenever } x>b .\] Dado que\(f\) es continuo en\([a, b]\), se limita en este intervalo. Por lo tanto,\(f\) está acotado en\([a, \infty]\).
    2. Fijar cualquiera\(\varepsilon > 0\), por la definición de límite, encontramos\(b > a\) tal que\[|f(x)-c|<\frac{\varepsilon}{2} \text { whenever } x>b .\] Dado que\(f\) es continuo en\([a, b + 1]\), es uniformemente continuo en este intervalo. Así, existe\(0 < \delta < 1\) tal que\[|f(u)-f(v)|<\frac{\varepsilon}{2} \text { whenever }|u-v|<\delta, u, v \in[a, c+1] .\] Entonces podemos demostrarlo\(|f(u)-f(v)|<\frac{\varepsilon}{2} \text { whenever }|u-v|<\delta, u, v \in[a, c+1]\) cuando sea\(|u-v|<\delta, u, v \in[a, \infty)\).
    3. Ya que\(\lim _{x \rightarrow \infty} f(x)=c>f(a)\), existe\(b > a\) tal que\[f(x)>f(a) \text { whenever } x>b .\] Así,\[\inf \{f(x): x \in[a, \infty)\}=\inf \{f(x): x \in[a, b]\} .\] La conclusión se desprende del Teorema del Valor Extremo para la función\(f\) on\([a, b]\).

    Ejercicio\(3.7.4\).

    Contestar

    Ya que\(\inf \{f(x): x \in[a, \infty)\}=\inf \{f(x): x \in[a, b]\}\), existe\(a > 0\) tal que\[f(x) \geq f(0) \text { whenever }|x|>a .\] Since\(f\) es menor semicontinuo, por el Teorema 3.7.3, tiene un mínimo\([ -a, a]\) absoluto encendido en algún momento\(\bar{x} \in [-a, a]\). Obviamente,\[f(x) \geq f(\bar{x}) \text { for all } x \in[-a, a] .\] en particular,\(f(0) \geq f(\bar{x})\). Si\(|x| > a\), entonces\[f(x) \geq f(0) \geq f(\bar{x}) .\] Por lo tanto,\(f\) tiene un mínimo absoluto en\(\bar{x}\).

    Observe que en esta solución, podemos usar cualquier número\(\gamma\) en el rango de en\(f\) lugar de\(f(0)\). Dado que cualquier función continua también es semicontinua inferior, el resultado de este problema es aplicable para funciones continuas. Por ejemplo, podemos usar este teorema para probar que cualquier polinomio con grado par tiene un mínimo absoluto on\(\mathbb{R}\). Dado que no\(\mathbb[R}\) es un conjunto compacto, no podemos usar el teorema del valor extremo directamente.TST9


    This page titled 5.3: CAPÍTULO 3 is shared under a CC BY-NC-SA license and was authored, remixed, and/or curated by Lafferriere, Lafferriere, and Nguyen (PDXOpen: Open Educational Resources) .